Indukcja matematyczna

Ze względu na specyfikę metody - osobny dział.
Bartekk02
Użytkownik
Użytkownik
Posty: 1
Rejestracja: 16 cze 2021, o 13:03
Płeć: Mężczyzna
wiek: 19

Indukcja matematyczna

Post autor: Bartekk02 »

Mam kłopot z przeprowadzeniem dowodu indukcyjnego takiego wzoru:

\(\displaystyle{ \sum_{i=1}^{n} x_i \cdot y_i = x_n \cdot \sum_{i=1}^{n} y_i - \sum_{i=1}^{n-1} ( x_{i+1} - x_i) \sum_{k=1}^{i} y_k }\)

Udało mi się sprawdzić dla \(\displaystyle{ n=1}\)

\(\displaystyle{ L = \sum_{i=1}^{1} x_i \cdot y_i = x_1 \cdot y_1 }\)

\(\displaystyle{ P = x_1 \cdot \sum_{i=1}^{1} y_i - \sum_{i=1}^{0} ( x_{i+1} - x_i) \sum_{k=1}^{i} y_k = x_1 \cdot y_1 }\)

czyli \(\displaystyle{ L = P}\)

Następnie zakładamy, że dowód zachodzi dla \(\displaystyle{ n}\) i sprawdzamy dla \(\displaystyle{ n+1}\)

\(\displaystyle{ L = \sum_{i=1}^{n+1} x_i \cdot y_i = x_{n+1} \cdot y_{n+1}\sum_{i=1}^{n} x_i \cdot y_i }\)

\(\displaystyle{ P = x_{n+1} \cdot \sum_{i=1}^{n+1} y_i - \sum_{i=1}^{n} ( x_{i+1} - x_i) \sum_{k=1}^{i} y_k = x_{n+1} \cdot y_{n+1}\sum_{i=1}^{n} x_i \cdot y_i - \sum_{i=1}^{n} ( x_{i+1} - x_i) \sum_{k=1}^{i} y_k }\)

Dalej mam kłopot. Mógłby mi ktoś pomóc?
Jan Kraszewski
Administrator
Administrator
Posty: 34126
Rejestracja: 20 mar 2006, o 21:54
Płeć: Mężczyzna
Lokalizacja: Wrocław
Podziękował: 3 razy
Pomógł: 5192 razy

Re: Indukcja matematyczna

Post autor: Jan Kraszewski »

Bartekk02 pisze: 16 cze 2021, o 13:47\(\displaystyle{ L = \sum_{i=1}^{n+1} x_i \cdot y_i =\red{ x_{n+1} \cdot y_{n+1}\sum_{i=1}^{n} x_i \cdot y_i } }\)

\(\displaystyle{ P = x_{n+1} \cdot \sum_{i=1}^{n+1} y_i - \sum_{i=1}^{n} ( x_{i+1} - x_i) \sum_{k=1}^{i} y_k = \red{x_{n+1} \cdot y_{n+1}\sum_{i=1}^{n} x_i \cdot y_i - \sum_{i=1}^{n} ( x_{i+1} - x_i) \sum_{k=1}^{i} y_k} }\)

Dalej mam kłopot.
Masz kłopot, bo oba czerwone zapisy są niepoprawne.

JK
ODPOWIEDZ